Roots of a quadratic

Algebra Level 3

Let α \alpha and β \beta be the roots of the equation x 2 + x 3 = 0 x^2 + x - 3 = 0 such that α < β \alpha < \beta . Find the value of the expression 4 β 2 α 3 4 \beta^2 - \alpha^3 .


The answer is 19.

This section requires Javascript.
You are seeing this because something didn't load right. We suggest you, (a) try refreshing the page, (b) enabling javascript if it is disabled on your browser and, finally, (c) loading the non-javascript version of this page . We're sorry about the hassle.

3 solutions

Tapas Mazumdar
Mar 5, 2017

By Vieta's formula, we have

α + β = 1 \alpha + \beta = -1

Note that

4 β 2 α 3 = 4 ( 1 α ) 2 α 3 = α 3 + 4 α 2 + 8 α + 4 = α ( α 2 + α 3 ) 0 + 5 ( α 2 + α ) = 3 + 4 = 5 ( 3 ) + 4 = 19 \begin{aligned} 4 \beta^2 - \alpha^3 &= 4 {(-1-\alpha)}^2 - \alpha^3 \\ &= -\alpha^3 + 4\alpha^2 + 8 \alpha +4 \\ &= -\alpha \cancel{(\alpha^2 + \alpha -3)}^{0} + 5 \underbrace{(\alpha^2 + \alpha)}_{= 3} + 4 \\ &= 5(3) + 4 \\ &= 19 \end{aligned}

Nice solution buddy! B)

Sahil Silare - 4 years, 3 months ago

x 2 x \cancel{x^2-x} , just trying latex, dont worry

Md Zuhair - 4 years, 1 month ago

Log in to reply

That command is to be used like this

\require{cancel} \cancel{x}

Which will give

x \cancel{x}

'\require{cancel}' command is important here.


Moreover

\require{cancel} \cancel{x}^1

will give

x 1 \cancel{x}^1

and

\require{cancel} \cancel{x}_1

will give

x 1 \cancel{x}_1

Tapas Mazumdar - 4 years, 1 month ago

BRUTE FORCE SOLUTION

By the quadratic formula, the roots of x 2 + x 3 x^2+x-3 , which will be denoted by α \alpha and β \beta , where α < β \alpha<\beta , are

α = 1 13 2 β = 1 + 13 2 . \begin{aligned} \alpha &= \frac{-1-\sqrt{13}}{2} \\ \beta &= \frac{-1+\sqrt{13}}{2}. \end{aligned}

So,

4 β 2 α 3 = 4 × ( 1 + 13 ) 2 4 ( 1 13 ) 3 8 = ( 1 + 13 ) 2 ( 1 13 ) 3 8 . \begin{aligned} 4\beta^2 - \alpha^3 &= 4 \times \frac{(-1+\sqrt{13})^2}{4} - \frac{(-1-\sqrt{13})^3}{8} \\ &= (-1+\sqrt{13})^2 - \frac{(-1-\sqrt{13})^3}{8}. \end{aligned}

Observe that

( 1 + 13 ) 2 = 14 2 13 ( 1 13 ) 3 = ( 14 + 2 13 ) ( 1 13 ) = 40 16 13 = 8 ( 5 + 2 13 ) . \begin{aligned} (-1+\sqrt{13})^2 &= 14 - 2\sqrt{13} \\ (-1-\sqrt{13})^3 &= (14+2\sqrt{13})(-1-\sqrt{13}) \\ &= -40-16\sqrt{13} = -8(5+2\sqrt{13}). \end{aligned}

As a result,

4 β 2 α 3 = 14 2 13 8 ( 5 + 2 13 ) 8 = 14 2 13 + 5 + 2 13 = 19 , \begin{aligned} 4\beta^2 - \alpha^3 &= 14 - 2\sqrt{13} - \frac{-8(5+2\sqrt{13})}{8} \\ &= 14 - 2\sqrt{13} + 5 + 2\sqrt{13} \\ &= 19, \end{aligned}

as required.

Its quire tough to do it like this

Md Zuhair - 4 years, 1 month ago

Log in to reply

Brute force solutions may not be the best way of arriving to solutions to many people, but it is a foolproof way of proving things. If you take issue with it, that's your issue, not mine's.

A Former Brilliant Member - 4 years, 1 month ago
Skye Rzym
Apr 12, 2017

From Vieta's Theorem, we get

β + α = 1 \beta + \alpha = -1

β α = 3 \beta\alpha = -3

And then, because α < β \alpha < \beta , so

β α = Discriminant \beta - \alpha = \sqrt{\text{Discriminant}}

β α = ( 1 ) 2 4.1. ( 3 ) \beta - \alpha = \sqrt{(-1)^{2}-4.1.(-3)}

β α = 1 + 4.3 = 13 \beta - \alpha = \sqrt{1+4.3}=\sqrt{13}

Now, let

x = 4 β 2 α 3 x = 4\beta^{2} - \alpha^{3} , and

y = 4 α 2 β 3 y = 4\alpha^{2} - \beta^{3}

Now

(1)...

x + y = 4 β 2 α 3 + 4 α 2 β 3 x+y = 4\beta^{2} - \alpha^{3} + 4\alpha^{2} - \beta^{3} x + y = 4 ( β 2 + α 2 ) ( α 3 + β 3 ) x+y = 4(\beta^{2} + \alpha^{2}) - (\alpha^{3} + \beta^{3}) x + y = 4 ( ( β + α ) 2 2 β α ) ( α + β ) ( α 2 β α + β 2 ) x+y = 4((\beta + \alpha)^{2} - 2\beta\alpha) - (\alpha + \beta)(\alpha^{2} - \beta\alpha + \beta^{2}) x + y = 4 ( ( 1 ) 2 2 ( 3 ) ) ( 1 ) ( ( α + β ) 2 3 α β ) x+y = 4((-1)^{2} - 2(-3)) - (-1)((\alpha + \beta)^{2} - 3\alpha\beta) x + y = 4 ( 1 + 2.3 ) + ( ( 1 ) 2 3 ( 3 ) ) x+y = 4(1 + 2.3) + ((-1)^{2} - 3(-3)) x + y = 4.7 + ( 1 + 3.3 ) = 28 + 10 = 38 x+y = 4.7 + (1 + 3.3) = 28 + 10 = 38

And then

(2)...

x y = 4 β 2 α 3 4 α 2 + β 3 x-y = 4\beta^{2} - \alpha^{3} - 4\alpha^{2} + \beta^{3} x y = 4 ( β 2 α 2 ) + ( β 3 α 3 ) x-y = 4(\beta^{2} - \alpha^{2}) + (\beta^{3} - \alpha^{3}) x y = 4 ( β + α ) ( β α ) + ( β α ) ( α 2 + β α + β 2 ) x-y = 4(\beta + \alpha)(\beta - \alpha) + (\beta - \alpha)(\alpha^{2} + \beta\alpha + \beta^{2}) x y = 4 ( ( 1 ) ( 13 ) + ( 13 ) ( ( α + β ) 2 α β ) x-y = 4((-1)(\sqrt{13}) + (\sqrt{13})((\alpha + \beta)^{2} - \alpha\beta) x y = 4 13 + 13 ( ( 1 ) 2 ( 3 ) ) x-y = -4\sqrt{13} + \sqrt{13}((-1)^{2} - (-3)) x y = 4 13 + 13 ( 1 + 3 ) = 4 13 + 4 13 = 0 x-y = -4\sqrt{13} + \sqrt{13}(1 + 3) = -4\sqrt{13} + 4\sqrt{13} = 0

Now, we find the value of x x .

(1)+(2)

x + y + x y = 38 + 0 x+y+x-y = 38+0 2 x = 38 2x = 38 x = 19 x = 19 4 β 2 α 3 = 19 4\beta^{2} - \alpha^{3} = \boxed{19}

Let suppose for a general quadratic a x 2 + b x + c = 0 ax^2+bx+c=0 whose roots are α \alpha and β \beta with α < β \alpha < \beta , so we have

β = b + b 2 4 a c 2 a α = b b 2 4 a c 2 a β α = 2 ( b 2 4 a c 2 a ) = Discriminant a \beta = \dfrac{-b + \sqrt{b^2 - 4ac}}{2a} \\ \alpha = \dfrac{-b - \sqrt{b^2 - 4ac}}{2a} \\ \implies \beta - \alpha = 2 \left( \dfrac{\sqrt{b^2-4ac}}{2a} \right) = \dfrac{\sqrt{\text{Discriminant}}}{a}

But as in the third step of your solution, you've simply stated β α = Discriminant \beta - \alpha = \sqrt{\text{Discriminant}} which may give a false information to someone reading your solution who doesn't have a good understanding of quadratics yet.

Tapas Mazumdar - 4 years, 1 month ago

0 pending reports

×

Problem Loading...

Note Loading...

Set Loading...